kmewmewblue
Thanks Received: 1
Forum Guests
 
Posts: 57
Joined: April 18th, 2011
 
 
trophy
Most Thankful
 

Q18 - Politician: A government that taxes

by kmewmewblue Thu Nov 03, 2011 11:52 pm

What is "limiting case" in this answer C?
If I take this phrase "if applied in the limiting case", this will not be an answer?
User avatar
 
bbirdwell
Thanks Received: 864
Atticus Finch
Atticus Finch
 
Posts: 803
Joined: April 16th, 2009
 
This post thanked 2 times.
 
 

Re: Q18 - Politician: A government that taxes

by bbirdwell Wed Nov 09, 2011 10:44 pm

"Limiting case" just means "in the specific situation mentioned." In this example, the limiting case was a tax rate of 0%. It's just a modifier. Feel free to eliminate it from the sentence, and the answer still holds: the author derives a principle and decides that it's absurd.
I host free online workshop/Q&A sessions called Zen and the Art of LSAT. You can find upcoming dates here: http://www.manhattanlsat.com/zen-and-the-art.cfm
 
wuqq577
Thanks Received: 0
Forum Guests
 
Posts: 1
Joined: September 08th, 2013
 
 
 

Re: Q18 - Politician: A government that taxes

by wuqq577 Sun Sep 08, 2013 12:21 pm

But what's the difference between choice B and C?
User avatar
 
maryadkins
Thanks Received: 641
Atticus Finch
Atticus Finch
 
Posts: 1261
Joined: March 23rd, 2011
 
This post thanked 1 time.
 
 

Re: Q18 - Politician: A government that taxes

by maryadkins Tue Sep 10, 2013 1:33 pm

The economist's strategy here is to apply the politician's principle--that the lower the tax rate, the more revenue will be generated--to a situation in which it would certainly not be true. If it were zero, there'd be no revenue at all.

(A) is incorrect because the economist isn't stating a general principle.

(B) is incorrect because the economist says nothing about "where the politician's advice has been adopted"--he says it "would mean" no revenue in a hypothetical case. As far as we know, the politician's advice hasn't been adopted anywhere.

(C) is correct.

(D) is incorrect; the economist doesn't question the politician's knowledge of economics.

(E) is incorrect because there isn't attack on a premise's truth. There's just application of the principle to an extreme case to show its absurdity.
 
roflcoptersoisoi
Thanks Received: 0
Atticus Finch
Atticus Finch
 
Posts: 165
Joined: April 30th, 2015
 
 
 

Re: Q18 - Politician: A government that taxes

by roflcoptersoisoi Sun Jul 10, 2016 2:56 pm

maryadkins Wrote:The economist's strategy here is to apply the politician's principle--that the lower the tax rate, the more revenue will be generated--to a situation in which it would certainly not be true. If it were zero, there'd be no revenue at all.

(A) is incorrect because the economist isn't stating a general principle.

(B) is incorrect because the economist says nothing about "where the politician's advice has been adopted"--he says it "would mean" no revenue in a hypothetical case. As far as we know, the politician's advice hasn't been adopted anywhere.

(C) is correct.

(D) is incorrect; the economist doesn't question the politician's knowledge of economics.

(E) is incorrect because there isn't attack on a premise's truth. There's just application of the principle to an extreme case to show its absurdity.



But isn't the economist's conclusion a principle? The Politician's conclusion is a principle and the economist's conclusion is just the negation of the politician's conclusion so it follows that the economist's conclusion is a principle that is incompatible with the principle stated by the politician no?